Put R under double integral












6















When I use $iint_R f(x,y)dA$ the letter $R$ is to the right of the double integral sign. How to make it under the sign? This is a simple question but I couldn't find a related question.










share|improve this question




















  • 2





    @JouleV I managed to mess up the markdown quoting:-), I'll delete and repost the comment, thanks.

    – David Carlisle
    17 hours ago






  • 3





    you are using inline mathematics ($) the entire design of the layout for inline math is to make it fit within the normal line spacing of the text in a paragraph so limits move to subscript position, if you need the display style it it best to set it as a math display([...])

    – David Carlisle
    17 hours ago
















6















When I use $iint_R f(x,y)dA$ the letter $R$ is to the right of the double integral sign. How to make it under the sign? This is a simple question but I couldn't find a related question.










share|improve this question




















  • 2





    @JouleV I managed to mess up the markdown quoting:-), I'll delete and repost the comment, thanks.

    – David Carlisle
    17 hours ago






  • 3





    you are using inline mathematics ($) the entire design of the layout for inline math is to make it fit within the normal line spacing of the text in a paragraph so limits move to subscript position, if you need the display style it it best to set it as a math display([...])

    – David Carlisle
    17 hours ago














6












6








6








When I use $iint_R f(x,y)dA$ the letter $R$ is to the right of the double integral sign. How to make it under the sign? This is a simple question but I couldn't find a related question.










share|improve this question
















When I use $iint_R f(x,y)dA$ the letter $R$ is to the right of the double integral sign. How to make it under the sign? This is a simple question but I couldn't find a related question.







subscripts






share|improve this question















share|improve this question













share|improve this question




share|improve this question








edited 19 hours ago









JouleV

14.4k22664




14.4k22664










asked 19 hours ago









Haoran ChenHaoran Chen

25918




25918








  • 2





    @JouleV I managed to mess up the markdown quoting:-), I'll delete and repost the comment, thanks.

    – David Carlisle
    17 hours ago






  • 3





    you are using inline mathematics ($) the entire design of the layout for inline math is to make it fit within the normal line spacing of the text in a paragraph so limits move to subscript position, if you need the display style it it best to set it as a math display([...])

    – David Carlisle
    17 hours ago














  • 2





    @JouleV I managed to mess up the markdown quoting:-), I'll delete and repost the comment, thanks.

    – David Carlisle
    17 hours ago






  • 3





    you are using inline mathematics ($) the entire design of the layout for inline math is to make it fit within the normal line spacing of the text in a paragraph so limits move to subscript position, if you need the display style it it best to set it as a math display([...])

    – David Carlisle
    17 hours ago








2




2





@JouleV I managed to mess up the markdown quoting:-), I'll delete and repost the comment, thanks.

– David Carlisle
17 hours ago





@JouleV I managed to mess up the markdown quoting:-), I'll delete and repost the comment, thanks.

– David Carlisle
17 hours ago




3




3





you are using inline mathematics ($) the entire design of the layout for inline math is to make it fit within the normal line spacing of the text in a paragraph so limits move to subscript position, if you need the display style it it best to set it as a math display([...])

– David Carlisle
17 hours ago





you are using inline mathematics ($) the entire design of the layout for inline math is to make it fit within the normal line spacing of the text in a paragraph so limits move to subscript position, if you need the display style it it best to set it as a math display([...])

– David Carlisle
17 hours ago










2 Answers
2






active

oldest

votes


















12














Apart from JouleV's nice answer, you can use limits option to typeset the inline with equation with limits under the integral symbol.



documentclass{article}
usepackage{amsmath}
begin{document}
$iintlimits_a f(x,y) dA$
end{document}


to get:



enter image description here






share|improve this answer





















  • 3





    I'd say that iintlimits_R centers the R term far better than underset{R}{iint} does.

    – Mico
    16 hours ago








  • 1





    No upper limit should be used when limits follows a command for multiple integrals. You should also respect the OP’s preference for the differential.

    – egreg
    16 hours ago











  • @egreg I have updated my answer, thanks for the remark. Could you also briefly explain why no upper limit must be used?

    – Raaja
    16 hours ago






  • 2





    @Raaja Because they’re badly positioned.

    – egreg
    16 hours ago











  • @egreg Huhh, like that ;) thanks.

    – Raaja
    16 hours ago



















4














I don't think this is a good idea, but if you want to have it, you can use underset:



documentclass{standalone}
usepackage{amsmath}
begin{document}
$underset{R}{iint} f(x,y)dA$
end{document}


enter image description here






share|improve this answer



















  • 1





    Sorry, but this is not the best choice.

    – egreg
    16 hours ago






  • 1





    @egreg Yes, it may not be the best, but it surely is a bad one. I will never write like that in my documents

    – JouleV
    16 hours ago














Your Answer








StackExchange.ready(function() {
var channelOptions = {
tags: "".split(" "),
id: "85"
};
initTagRenderer("".split(" "), "".split(" "), channelOptions);

StackExchange.using("externalEditor", function() {
// Have to fire editor after snippets, if snippets enabled
if (StackExchange.settings.snippets.snippetsEnabled) {
StackExchange.using("snippets", function() {
createEditor();
});
}
else {
createEditor();
}
});

function createEditor() {
StackExchange.prepareEditor({
heartbeatType: 'answer',
autoActivateHeartbeat: false,
convertImagesToLinks: false,
noModals: true,
showLowRepImageUploadWarning: true,
reputationToPostImages: null,
bindNavPrevention: true,
postfix: "",
imageUploader: {
brandingHtml: "Powered by u003ca class="icon-imgur-white" href="https://imgur.com/"u003eu003c/au003e",
contentPolicyHtml: "User contributions licensed under u003ca href="https://creativecommons.org/licenses/by-sa/3.0/"u003ecc by-sa 3.0 with attribution requiredu003c/au003e u003ca href="https://stackoverflow.com/legal/content-policy"u003e(content policy)u003c/au003e",
allowUrls: true
},
onDemand: true,
discardSelector: ".discard-answer"
,immediatelyShowMarkdownHelp:true
});


}
});














draft saved

draft discarded


















StackExchange.ready(
function () {
StackExchange.openid.initPostLogin('.new-post-login', 'https%3a%2f%2ftex.stackexchange.com%2fquestions%2f485553%2fput-r-under-double-integral%23new-answer', 'question_page');
}
);

Post as a guest















Required, but never shown

























2 Answers
2






active

oldest

votes








2 Answers
2






active

oldest

votes









active

oldest

votes






active

oldest

votes









12














Apart from JouleV's nice answer, you can use limits option to typeset the inline with equation with limits under the integral symbol.



documentclass{article}
usepackage{amsmath}
begin{document}
$iintlimits_a f(x,y) dA$
end{document}


to get:



enter image description here






share|improve this answer





















  • 3





    I'd say that iintlimits_R centers the R term far better than underset{R}{iint} does.

    – Mico
    16 hours ago








  • 1





    No upper limit should be used when limits follows a command for multiple integrals. You should also respect the OP’s preference for the differential.

    – egreg
    16 hours ago











  • @egreg I have updated my answer, thanks for the remark. Could you also briefly explain why no upper limit must be used?

    – Raaja
    16 hours ago






  • 2





    @Raaja Because they’re badly positioned.

    – egreg
    16 hours ago











  • @egreg Huhh, like that ;) thanks.

    – Raaja
    16 hours ago
















12














Apart from JouleV's nice answer, you can use limits option to typeset the inline with equation with limits under the integral symbol.



documentclass{article}
usepackage{amsmath}
begin{document}
$iintlimits_a f(x,y) dA$
end{document}


to get:



enter image description here






share|improve this answer





















  • 3





    I'd say that iintlimits_R centers the R term far better than underset{R}{iint} does.

    – Mico
    16 hours ago








  • 1





    No upper limit should be used when limits follows a command for multiple integrals. You should also respect the OP’s preference for the differential.

    – egreg
    16 hours ago











  • @egreg I have updated my answer, thanks for the remark. Could you also briefly explain why no upper limit must be used?

    – Raaja
    16 hours ago






  • 2





    @Raaja Because they’re badly positioned.

    – egreg
    16 hours ago











  • @egreg Huhh, like that ;) thanks.

    – Raaja
    16 hours ago














12












12








12







Apart from JouleV's nice answer, you can use limits option to typeset the inline with equation with limits under the integral symbol.



documentclass{article}
usepackage{amsmath}
begin{document}
$iintlimits_a f(x,y) dA$
end{document}


to get:



enter image description here






share|improve this answer















Apart from JouleV's nice answer, you can use limits option to typeset the inline with equation with limits under the integral symbol.



documentclass{article}
usepackage{amsmath}
begin{document}
$iintlimits_a f(x,y) dA$
end{document}


to get:



enter image description here







share|improve this answer














share|improve this answer



share|improve this answer








edited 16 hours ago

























answered 19 hours ago









RaajaRaaja

5,44121645




5,44121645








  • 3





    I'd say that iintlimits_R centers the R term far better than underset{R}{iint} does.

    – Mico
    16 hours ago








  • 1





    No upper limit should be used when limits follows a command for multiple integrals. You should also respect the OP’s preference for the differential.

    – egreg
    16 hours ago











  • @egreg I have updated my answer, thanks for the remark. Could you also briefly explain why no upper limit must be used?

    – Raaja
    16 hours ago






  • 2





    @Raaja Because they’re badly positioned.

    – egreg
    16 hours ago











  • @egreg Huhh, like that ;) thanks.

    – Raaja
    16 hours ago














  • 3





    I'd say that iintlimits_R centers the R term far better than underset{R}{iint} does.

    – Mico
    16 hours ago








  • 1





    No upper limit should be used when limits follows a command for multiple integrals. You should also respect the OP’s preference for the differential.

    – egreg
    16 hours ago











  • @egreg I have updated my answer, thanks for the remark. Could you also briefly explain why no upper limit must be used?

    – Raaja
    16 hours ago






  • 2





    @Raaja Because they’re badly positioned.

    – egreg
    16 hours ago











  • @egreg Huhh, like that ;) thanks.

    – Raaja
    16 hours ago








3




3





I'd say that iintlimits_R centers the R term far better than underset{R}{iint} does.

– Mico
16 hours ago







I'd say that iintlimits_R centers the R term far better than underset{R}{iint} does.

– Mico
16 hours ago






1




1





No upper limit should be used when limits follows a command for multiple integrals. You should also respect the OP’s preference for the differential.

– egreg
16 hours ago





No upper limit should be used when limits follows a command for multiple integrals. You should also respect the OP’s preference for the differential.

– egreg
16 hours ago













@egreg I have updated my answer, thanks for the remark. Could you also briefly explain why no upper limit must be used?

– Raaja
16 hours ago





@egreg I have updated my answer, thanks for the remark. Could you also briefly explain why no upper limit must be used?

– Raaja
16 hours ago




2




2





@Raaja Because they’re badly positioned.

– egreg
16 hours ago





@Raaja Because they’re badly positioned.

– egreg
16 hours ago













@egreg Huhh, like that ;) thanks.

– Raaja
16 hours ago





@egreg Huhh, like that ;) thanks.

– Raaja
16 hours ago











4














I don't think this is a good idea, but if you want to have it, you can use underset:



documentclass{standalone}
usepackage{amsmath}
begin{document}
$underset{R}{iint} f(x,y)dA$
end{document}


enter image description here






share|improve this answer



















  • 1





    Sorry, but this is not the best choice.

    – egreg
    16 hours ago






  • 1





    @egreg Yes, it may not be the best, but it surely is a bad one. I will never write like that in my documents

    – JouleV
    16 hours ago


















4














I don't think this is a good idea, but if you want to have it, you can use underset:



documentclass{standalone}
usepackage{amsmath}
begin{document}
$underset{R}{iint} f(x,y)dA$
end{document}


enter image description here






share|improve this answer



















  • 1





    Sorry, but this is not the best choice.

    – egreg
    16 hours ago






  • 1





    @egreg Yes, it may not be the best, but it surely is a bad one. I will never write like that in my documents

    – JouleV
    16 hours ago
















4












4








4







I don't think this is a good idea, but if you want to have it, you can use underset:



documentclass{standalone}
usepackage{amsmath}
begin{document}
$underset{R}{iint} f(x,y)dA$
end{document}


enter image description here






share|improve this answer













I don't think this is a good idea, but if you want to have it, you can use underset:



documentclass{standalone}
usepackage{amsmath}
begin{document}
$underset{R}{iint} f(x,y)dA$
end{document}


enter image description here







share|improve this answer












share|improve this answer



share|improve this answer










answered 19 hours ago









JouleVJouleV

14.4k22664




14.4k22664








  • 1





    Sorry, but this is not the best choice.

    – egreg
    16 hours ago






  • 1





    @egreg Yes, it may not be the best, but it surely is a bad one. I will never write like that in my documents

    – JouleV
    16 hours ago
















  • 1





    Sorry, but this is not the best choice.

    – egreg
    16 hours ago






  • 1





    @egreg Yes, it may not be the best, but it surely is a bad one. I will never write like that in my documents

    – JouleV
    16 hours ago










1




1





Sorry, but this is not the best choice.

– egreg
16 hours ago





Sorry, but this is not the best choice.

– egreg
16 hours ago




1




1





@egreg Yes, it may not be the best, but it surely is a bad one. I will never write like that in my documents

– JouleV
16 hours ago







@egreg Yes, it may not be the best, but it surely is a bad one. I will never write like that in my documents

– JouleV
16 hours ago




















draft saved

draft discarded




















































Thanks for contributing an answer to TeX - LaTeX Stack Exchange!


  • Please be sure to answer the question. Provide details and share your research!

But avoid



  • Asking for help, clarification, or responding to other answers.

  • Making statements based on opinion; back them up with references or personal experience.


To learn more, see our tips on writing great answers.




draft saved


draft discarded














StackExchange.ready(
function () {
StackExchange.openid.initPostLogin('.new-post-login', 'https%3a%2f%2ftex.stackexchange.com%2fquestions%2f485553%2fput-r-under-double-integral%23new-answer', 'question_page');
}
);

Post as a guest















Required, but never shown





















































Required, but never shown














Required, but never shown












Required, but never shown







Required, but never shown

































Required, but never shown














Required, but never shown












Required, but never shown







Required, but never shown







Popular posts from this blog

How did Captain America manage to do this?

迪纳利

南乌拉尔铁路局